Diễn Đàn MathScopeDiễn Đàn MathScope
  Diễn Đàn MathScope
Ghi Danh Hỏi/Ðáp Community Lịch

Go Back   Diễn Đàn MathScope > Sơ Cấp > Tài Liệu > Đề Thi > Đề Thi HSG Cấp Tỉnh ở Việt Nam

News & Announcements

Ngoài một số quy định đã được nêu trong phần Quy định của Ghi Danh , mọi người tranh thủ bỏ ra 5 phút để đọc thêm một số Quy định sau để khỏi bị treo nick ở MathScope nhé !

* Nội quy MathScope.Org

* Một số quy định chung !

* Quy định về việc viết bài trong diễn đàn MathScope

* Nếu bạn muốn gia nhập đội ngũ BQT thì vui lòng tham gia tại đây

* Những câu hỏi thường gặp

* Về việc viết bài trong Box Đại học và Sau đại học


Trả lời Gởi Ðề Tài Mới
 
Ðiều Chỉnh Xếp Bài
Old 13-10-2017, 07:30 PM   #1
hoanganhtran
Moderator
 
Tham gia ngày: Jan 2017
Bài gởi: 8
Thanks: 1
Thanked 3 Times in 3 Posts
$\boxed{21}$ [Quảng Ninh] Tìm tất cả các số nguyên tố $p$ sao cho $\dfrac{3^{p-1}-1}{p}$ là một số chính phương.
Bài giải
Giả sử tồn tại số nguyên tố $p$ thỏa mãn $\dfrac{3^{p-1}-1}{p}$ là một số chính phương. Khi đó tồn tại số tự nhiên $A$ sao cho
$3^{p-1}-1=p.A^2$
Nếu $p=2$ thì $3^{p-1}-1=p.A^2$ hiển nhiên đúng.
Nếu $p > 2$ và $p$ là số nguyên tố nên hiển nhiên $p$ lẻ. Đặt $p-1=2.k$ với $k$ nguyên dương. Khi đó
$3^{2k}-1=p.A^2$ mà $(3^{k}-1,3^{k}+1)=2$ nên tồn tại các số nguyên dương $B$ và $C$ sao cho:
hoặc $3^{k}-1=2pB^2$, $3{k}+1=2C^2$
hoặc $3^{k}-1=2.C^2$, $3^{k}+1=2pB^2$

Trường hợp thứ nhất không thể xảy ra do nếu $3^{k}+1=2C^2$ thì $2.C^2 \equiv1(mod 3)$. Điều này không xảy ra do $C^2$ chỉ có thể đồng dư với 0 hoặc 1 modunlo 3
Trường hợp 2:$3^{k}-1=2.C^2$, $3^{k}+1=2pB^2$
Nếu $k$ lẻ thì theo tính chất bình phương của 1 số nguyên lẻ luôn đồng dư với 1 modunlo 8 nên $3^{k}+1 \equiv 4 (mod 8)$ từ đó suy ra $2.C^2 \equiv 4 (mod 8)$ suy ra $C^2 \equiv 2 (mod 4)$. Điều này là vô lí do 1 số chính phương chỉ đồng dư 0 hoặc 1 modunlo 4
Từ lập luận trên ta có $k$ là số chẵn. Đặt $k=2.m$ với $m$ là số tự nhiên
$$3^{2.m}-1=2.C^2$$ nên hoặc $3^{m}-1=D^2$, $3^{m}+1=2.E^2$
hoặc $3^{m}-1=2.D^2$, $3^{m}+1=E^2$
trong đó $D$ và $E$ là các số nguyên dương và $D.E=C$
Trường hợp thứ nhất, do $D^2$ chỉ có thể đồng dư 0 hoặc 1 modunlo 3 nên $m=0$ hay $k=0$ suy ra $p=1$ vô lí.
Trường hợp 2: $3^{m}-1=2.D^2$, $3^{m}+1=E^2$ nên $3^{m}=(E-1)(E+1)$ suy ra $E-1$ và $E+1$ đều là lũy thừa của 3, mặt khác $(E-1,E+1)=2$ không chia hết cho 3 nên $E-1=1$ hay $3^{m}=3$ nên
$m=1$ suy ra $p=5$. Thử lại thỏa mãn đề bài.
Vậy $p=2$ và $p=5$ là tất cả các số nguyên tố thỏa mãn đề bài.
[RIGHT][I][B]Nguồn: MathScope.ORG[/B][/I][/RIGHT]
 
hoanganhtran is offline   Trả Lời Với Trích Dẫn
The Following User Says Thank You to hoanganhtran For This Useful Post:
2M (13-10-2017)
Old 22-10-2017, 04:27 PM   #2
hoanganhtran
Moderator
 
Tham gia ngày: Jan 2017
Bài gởi: 8
Thanks: 1
Thanked 3 Times in 3 Posts
$ HÀ NỘI $ Cho $x$ $y$ $z$ là các số hữu tỉ thỏa mãn $x+y^2+z^2$ , $y+x^2+z^2$ $z+y^2+x^2$ là các số nguyên. CMR 2x là số nguyên.
Lời gải:
Giả sử tồn tại các số hữu tỉ $x$ $y$ $z$ thỏa mãn đề bài. khi đó tồn tại các số nguyên $a$ $b$ $c$ $d$ thỏa mãn $x= \dfrac {a}{d}$, $y= \dfrac {b}{d}$, $z= \dfrac {c}{d}$ và $(a,b,c,d)=1$.
Giả thiết sẽ tương đương với $ ad+b^2+c^2 \equiv bd+a^2+c^2 \equiv cd+a^2+b^2 \equiv 0 (mod d^2)$ (1)
Giả sử d có ước nguyên tố lẻ là $p$ thì từ (1) ta có $a^2+b^2 \equiv b^2+c^2 \equiv a^2+c^2 \equiv 0 (mod p) \Rightarrow a^2 \equiv b^2 \equiv c^2 (mod p)$
Suy ra $2a^2$ $2b^2$ $2c^2$ chia hết cho $p$ mà $p$ là số nguyên tố lẻ nên $a$ $b$ $c$ đều chia hết cho p nên $(a,b,c,d) \equiv 0 (mod p)$ (mâu thuẫn với $(a,b,c,d)=1$ )
Vậy d không có ước nguyên tố hay $p$ hay d là lũy thừa của 2. Đặt $d=2^m$ ($m$ là số tự nhiên)
Giả sử $m \geq 2$ thì $a^2+b^2$ $b^2+c^2$ $ a^2+c^2$ chia hết cho 4 nên a,b,c phải cùng chẵn( do bình phương của 1 số lẻ luôn đồng dư với 1 modun 4) (mâu thuẫn với $(a,b,c,d)=1$)
Vậy $m<2$ hay d là ước của 2 suy ra $2x$ là số nguyên(đpcm).
[RIGHT][I][B]Nguồn: MathScope.ORG[/B][/I][/RIGHT]
 
hoanganhtran is offline   Trả Lời Với Trích Dẫn
The Following User Says Thank You to hoanganhtran For This Useful Post:
thepduc (22-10-2017)
Old 13-10-2017, 09:37 PM   #3
hoanganhtran
Moderator
 
Tham gia ngày: Jan 2017
Bài gởi: 8
Thanks: 1
Thanked 3 Times in 3 Posts
$\boxed{21}$ [Quảng Ninh] Tìm tất cả các số nguyên tố $p$ sao cho $\dfrac{3^{p-1}-1}{p}$ là một số chính phương.
Tiếp tục với bài toán khi ta thay 3 bởi 5.
Tìm tất cả các số nguyên tố $p$ sao cho $\dfrac{5^{p-1}-1}{p}$ là một số chính phương.
Lời giải
Giả sử tồn tại số nguyên tố $p$ thỏa mãn đề bài. Gọi số tự nhiên $A$ là số thỏa mãn $5^{p-1}-1=p.A^2$.
Nếu $p=2$ thì $\dfrac{5^{p-1}-1}{p}=2$ không phải là số nguyên tố.
Nếu $p>2$ thì $p-1$ lẻ nên tồn tại số tự nhiên $k$ sao cho $p-1=2k$.
Khi đó $5^{2k}-1=(5^{k}-1)(5^{k}+1)=pA^2$ và $(5^{k}-1,5^{k}+1)=1$ nên tồn tại các số tự nhiên $B$ và $C$ thỏa
hoặc $$5^{k}-1=2B^2, 5^{k}+1=2pC^2$$
hoặc $$5^{k}-1=2pB^2,5^{k}+1=2C^2$$
Do 1 số chính phương chia 5 dư 0,1,4 nên cả 2 th ko thể xảy ra. Trong th1 thì $5^{k}-1 \equiv 4 \equiv 2B^2 (mod 5)$ từ đó suy ra $B^2$ đòng dư 2 mod 5, vô lí.
Trong th2 thì $5^{k}+1 \equiv 6 \equiv 2C^2 (mod 5)$ nên $C^2 \equiv 3 (mod 5)$, vô lí.
Vậy với số 5 không thỏa mãn số nguyên tố $p$ thỏa mãn đề bài.
Tiếp tục thay số 5 bởi số 7, ta có bài toán:
Tìm tất cả các số nguyên tố $p$ sao cho $\dfrac{7^{p-1}-1}{p}$ là một số chính phương.
Lời giải
Vẫn với lâp luận như trên.
Nếu $p=2$ thì $\dfrac{7^{p-1}-1}{p}=3$ không phải là 1 số chính phương.
Nếu $p>2$ thì vẫn tồn tại cac số tự nhiên $k$ $B$ $C$ thỏa $p-1=2k$ và:
hoặc $$7^{k}-1=2pB^2, 7^{k}+1=2C^2$$ hoặc $$7^{k}-1=2pB^2, 7^{k}+1=2pC^2$$
Do 1 số chính phương chia 7dư 0,1,2,4 nên trường hợp 2 không xảy ra.
Trường hợp 1:ta có $7^{k}-1$ chia hết cho 3 nên $pB^2$ phải chia hết cho 3 từ đó suy ra hoặc $p=3$ hoặc $B$ chia hết cho 3.
Hiển nhiên thấy $p=3$ thỏa mãn đề bài. Xét đến trưởng hợp $B$ chia hết cho 3 nên $pB^2$ chia hết cho 9 nên $7^{k}-1$ chia hết cho 9 suy ra k chia hết cho 3.
Đặt $k=3m$ và $x=7^{m}$ thì ta có $x^3+1=2C^2$. Hiển nhiên $x+1$ không chia hết cho 3 nên $(x+1,x^2-x+1)=1$ nên x^2-x+1 phải là số chính phương. Điều này không xảy ra.
Vậy $p=3$ là số nguyên tốt duy nhất thỏa mãn đề bài.
[RIGHT][I][B]Nguồn: MathScope.ORG[/B][/I][/RIGHT]
 
hoanganhtran is offline   Trả Lời Với Trích Dẫn
Old 15-10-2017, 01:30 AM   #4
hoanganhtran
Moderator
 
Tham gia ngày: Jan 2017
Bài gởi: 8
Thanks: 1
Thanked 3 Times in 3 Posts
Mở rộng bài toán số học trong đề thi chọn hsg của Quảng Ninh.

Đề bài:Tìm tất cả các cặp số nguyên dương $p$ sao cho $p$ là số nguyên tố và tồn tại số nguyên dương a sao cho $\dfrac {(2a)^{p-1}-1}{p}$ là số chính phương.
Lời giải:
Giả sử tồn tại cặp số nguyên dương $(a,p)$ thỏa mã đề bài. Khi đó sẽ tồn tại số nguyên dương $A$ sao cho $(2a)^{p-1}-1=pA^2$
Do $(2a)^{p-1}-1$ là số lẻ nên $p$ cũng là số lẻ nên tồn tại số tự nhiên $m$ sao cho $p-1=2m$. Ta có $$(2a)^{2m}-1=pA^2$$
Hiển nhiên $((2a)^{m}-1,(2a)^{m}+1)=1$ nên tồn tại các số nguyên dương $B$ và $C$ sao cho hoặc $$(2a)^{m}-1=B^2,(2a)^{m}+1=pC^2$$
hoặc
$$(2a)^{m}-1=pB^2,(2a)^{m}+1=C^2$$
Trường hợp 1: $$(2a)^{m}-1=B^2,(2a)^{m}+1=pC^2$$
Ta có $(2a)^{m}-1$ là số lẻ nên $B \equiv 1 (mod 2) \Rightarrow B^2 \equiv 1 (mod 4) \Rightarrow (2a)^{m} \equiv 2 (mod 4) \Rightarrow m=1 \Rightarrow p=3$
Chọn a bằng 1 thì $4-1=3.1^2$ hoặc ta cũng có cách tìm ra tất cả các số a thảo mãn như sau:
Ta có $(2a)^2-1=3A^2$ mà $(2a)-1,(2a)+1)=1$ nên tồn tại các số nguyên dương $D$ và $E$ sao cho
hoặc $$2a=D^2+1=3E^2-1$$
hoặc $$2a=D^2-1=3E^2+1$$
Nếu $$2a=D^2+1=3E^2-1$$
thì $D$ và $E$ đều là các số lẻ nên $D+E$ và $D+3E$ đều lẻ và ta có $( \dfrac {D+3E}{2})^2-3( \dfrac {D+E}{2})^2=1$ là phương trình pell loại 1 và $(2,1)$ là bộ nghiệm nguyên dương nhỏ nhất của phương trình pell $x^2-3y^2=1$ nên
$$ \begin{cases}
\dfrac {D+3E}{2}= \frac {(2+ \sqrt{3})^n+(2- \sqrt{3})^n}{2} \\
\dfrac {D+E}{2}= \frac {(2+ \sqrt{3})^n-(2- \sqrt{3})^n}{2 \sqrt{3}}
\end{cases}$$ nên $$E=\frac {(2+ \sqrt{3})^n+(2- \sqrt{3})^n}{2}-\frac {(2+ \sqrt{3})^n-(2- \sqrt{3})^n}{2 \sqrt{3}}$$ mặt khác do $2a=3E^2+1$ nên $E$ lẻ từ đó suy ra n chẵn hay $E=\frac {(2+ \sqrt{3})^{2n}+(2- \sqrt{3})^{2n}}{2}-\frac {(2+ \sqrt{3})^{2n}-(2- \sqrt{3})^{2n}}{2 \sqrt{3}} \Rightarrow a=3( \frac {(2+ \sqrt{3})^{2}+(2- \sqrt{3})^{2n}}{2n}-\frac {(2+ \sqrt{3})^{2n}-(2- \sqrt{3})^{2n}}{2 \sqrt{3}})^2-1$ với n là số tự nhiên.
Nếu $$2a=D^2-1=3E^2+1$$ Trường hợp này không xảy ra do 1 số chính phương chỉ có thể đồng dư 1 mod 3 mà $D^2 \equiv 2 (mod 3)$.
Trường hợp 2: $$(2a)^{m}-1=pB^2,(2a)^{m}+1=C^2$$ mà $(C-1,C+1)=2$ nên tồn tại các số nguyên dương $y$ và $z$ sao cho
hoặc $$C-1=2y^{m}; C+1=2^{t}.z^{m}$$
hoặc $$C+1=2y^{m}; C-1=2^{t}.z^{m}$$ trong đó số nguyên t thỏa mãn $t+1=V_{2}((2a)^{m})=m.V_{2}(2a)$
Suy ra $y^{m}-2^{t-1}.z^{m}= \pm 1$
Hiển nhiên $(a,p)=(y,p)=(z,p)=1$ và $m= \dfrac {p-1}{2}$ nên $y^{m} \equiv \pm 1 (mod p)$ và $z^{m} \equiv \pm 1 (mod p)$
Từ đó ta có hoặc $$2^{t-1}-1 \equiv 1 (mod p)$$ hoặc $$2^{t-1}+1 \equiv -1 (mod p)$$ hoặc $$2^{t-1}+1 \equiv 1 (mod p) \Rightarrow p=2(vô lí) $$ hoặc $$2^{t-1}-1 \equiv -1 (mod p) \Rightarrow p=2(vô lí)$$
Kết hợp các trường hợp trên ta có $2^{2(t-1)}-1 \equiv 1 (mod p) \Rightarrow 2^{2(t+1)} \equiv 8 (mod p) \Rightarrow 1 \equiv 8 (mod p)$ (do $t+1= \dfrac {p-1}{2}.V_{2}(a) \Rightarrow 2(t+1) \equiv 0 (mod p)$ )
suy ra p=7, chọn a=1 thì ta có $2^6-1=7.3^2$
[RIGHT][I][B]Nguồn: MathScope.ORG[/B][/I][/RIGHT]
 

thay đổi nội dung bởi: hoanganhtran, 15-10-2017 lúc 03:53 AM Lý do: chưa hoàn thiện
hoanganhtran is offline   Trả Lời Với Trích Dẫn
Old 18-10-2017, 08:47 AM   #5
tikita
Administrator

 
Tham gia ngày: Jun 2012
Bài gởi: 157
Thanks: 2
Thanked 84 Times in 53 Posts
Trích:
Nguyên văn bởi MATHSCOPE View Post
Thời điểm này, nhiều tỉnh và các trường chuyên đã và đang hoàn tất việc thi chọn đội tuyển học sinh giỏi tham dự VMO. Tiếp nối truyền thống nhiều năm trước, www.mathscope.org mở lại chuyên mục này. Công việc này vừa là để các thầy cô và các bạn học sinh có một nguồn tư liệu tham khảo, vừa để thúc tiến các thảo luận của các thành viên Mathscope.

$\boxed{11}$ [Đồng Tháp] Xét tập hợp $S=\{1;\,2;\,\ldots;\,2017\}$. Ta tô màu mỗi phần tử của $S$ bởi một trong năm màu là Xanh, Đỏ, Tím, Vàng và Nâu. Chứng minh rằng tồn tại ba phần tử phân biệt $a;\,b;\,c$ của $S$ có cùng màu và thoả mãn $a\mid b$ và $b\mid c$.
Ta có $11$ số $1,2,2^2,2^3,2^4,2^5,2^6,2^7,2^8,2^9,2^{10}$ đều thuộc $S$ và chúng được tô bởi một trong năm màu, nên tồn tại ít nhất ba số sẽ được tô cùng màu, đây là ba số cần tìm và thỏa mãn điều kiện bài toán.
[RIGHT][I][B]Nguồn: MathScope.ORG[/B][/I][/RIGHT]
 
tikita is offline   Trả Lời Với Trích Dẫn
The Following User Says Thank You to tikita For This Useful Post:
2M (24-10-2017)
Old 19-10-2017, 12:47 AM   #6
312cr9
+Thành Viên+
 
Tham gia ngày: Aug 2014
Bài gởi: 10
Thanks: 3
Thanked 2 Times in 2 Posts
Trích:
Nguyên văn bởi MATHSCOPE View Post

$\boxed{8}$ [Hoà Bình] Tìm tất cả các cặp số nguyên dương $(a;\,b)$ sao cho$\left(a^2+b\right)\left(a+b^2\right)$ là một luỹ thừa của 2.
Nếu $a;\,b$ thoả yêu cầu thì sẽ phải tồn tại $m;\,n\in\mathbb Z^+$ thoả
\[\begin{array}{l}
{a^2} + b = {2^m}\\
{b^2} + a = {2^n}
\end{array}\]
Xét hai trường hợp sau
  1. Nếu $a\ne b$, ta giả sử $a>b$ và thấy
    \[{2^m} - {2^n} = \left( {a - b} \right)\left( {a + b - 1} \right)\]
    Như vậy, $m>n$ ta viết $m=n+k$ với $k\in\mathbb Z^+$ để có
    \[\left( {a - b} \right)\left( {a + b - 1} \right) = {2^n}\left( {{2^k} - 1} \right)\]
    Giờ để ý rằng $a;\,b$ có cùng tính chẵn lẻ cho nên $a+b-1$ lẻ tức là
    \[a-b\;\vdots\;2^n\]
    Điều này dẫn đến $a-b\ge 2^n=a+b^2$ tức $b^2+b\le 0$, và điều đó không thể xảy ra.
  2. Nếu $a=b$, thế thì $a(a+1)=2^m$. Tức $a$ và $a+1$ không được có ước nguyên tố nào ngoài 2. Mà $\gcd(a;\,a+1)=1$ và $a+1>1$ nên chỉ có thể $a=1$. Với $a=b=1$, ta thử thấy thoả.
Tóm lại $a=b=1$.
[RIGHT][I][B]Nguồn: MathScope.ORG[/B][/I][/RIGHT]
 
312cr9 is offline   Trả Lời Với Trích Dẫn
Old 19-10-2017, 04:41 PM   #7
Thụy An
+Thành Viên+

 
Tham gia ngày: Oct 2017
Bài gởi: 93
Thanks: 1
Thanked 68 Times in 45 Posts
Trích:
Nguyên văn bởi MATHSCOPE View Post
$\boxed{31}$ [Hoà Bình] Tìm các hàm số $f:\, \mathbb{R}\rightarrow \mathbb{R}$ thoả mãn
\[f\left( x \right) + f\left( y \right) + f\left( {xy} \right) = f\left( {x + y} \right) + f\left( x \right)f\left( y \right)\quad\forall\,x;\,y\in\mathbb R.\]
Gọi $P(x;\,y)$ là mệnh đề: $f\left( x \right) + f\left( y \right) + f\left( {xy} \right) = f\left( {x + y} \right) + f\left( x \right)f\left( y \right)$ đúng với $x;\,y$.

Từ $P(0;\,0)$ ta có $f(0)\in\{0;\,2\}$.
  1. Nếu $f(0)=2$, từ $P(x;\,0)$ có $f(x)=2\;\forall\,x\in\mathbb R$. Thử lại thấy nghiệm hàm này thoả mãn.
  2. Nếu $f(0)=0$, đặt $f(1)=k$ và từ $P(x;\,1)$ có
    \[f\left( {x + 1} \right) = \left( {2 - k} \right)f\left( x \right) + k\;\forall\,x\in\mathbb R\]
    Từ đó có
    \[\begin{array}{l}
    f\left( {x + 2} \right) &= \left( {2 - k} \right)f\left( {x + 1} \right) + k\\
    &= {\left( {2 - k} \right)^2}f\left( x \right) + 3k - {k^2}.
    \end{array}\]
    Thay $x=0$ vào để có
    \[f\left( 2 \right) = 3k - {k^2}\]
    Tức là có
    \[f\left( {x + 2} \right) = {\left( {2 - k} \right)^2}f\left( x \right) + f\left( 2 \right);\;(1).\]
    Lại từ $P(x;\,2)$ mà có
    \[f\left( {2x} \right) + f\left( x \right) + f\left( 2 \right) = f\left( {x + 2} \right) + f\left( x \right)f\left( 2 \right)\]
    Kết hợp $(1)$, đặt $3-k=l$ để có
    \[f(2x)=(3-k)f(x)=lf(x);\;(2).\]
    Từ $P(2x;\,2y)$ và $(2)$ có
    \[lf\left( {x + y} \right) + {l^2}f\left( x \right)f\left( y \right) = {l^2}f\left( {xy} \right) + lf\left( x \right) + lf\left( y \right);\;(3)\]
    • Nếu $l=0$ thì từ $(2)$ có $f(x)=0\;\forall\,x\in\mathbb R$. Thử lại thấy nghiệm hàm này thoả mãn.
    • Nếu $l=1$ tức $k=2$ thì từ $(1)$ có $f(x)=f(2)\;\forall\,x\in\mathbb R$. Tức $f(x)$ là hàm hằng và $f(x)=f(1)=2\;\forall\,x\in\mathbb R$.
      Thử lại thấy nghiệm hàm này thoả mãn.
    • Nếu $l\notin\{0;\,1\}$ thì từ $(3)$ và $P(x;\,y)$ có
      \[\begin{array}{l}
      l\left( {f\left( {xy} \right) - f\left( x \right)f\left( y \right)} \right) &= f\left( {x + y} \right) - f\left( x \right) - f\left( y \right)\\
      &= f\left( {xy} \right) - f\left( x \right)f\left( y \right)\quad\forall\,x;\,y\in\mathbb R.
      \end{array}\]
      Vậy nên có
      \[\begin{cases}
      f\left( {xy} \right) &= f\left( x \right)f\left( y \right)\\
      f\left( {x + y} \right) &= f\left( x \right) + f\left( y \right)
      \end{cases}\quad\forall\,x;\,y\in\mathbb R.\]
      Từ đây thấy $f$ cộng tính và tăng nên có nghiệm hàm $f(x)=x\;\forall\,x\in\mathbb R.$
Tóm lại, có ba nghiệm hàm thoả yêu cầu là $f_1(x)=0,\;f_2(x)=2$ và $f(x)=x$.
[RIGHT][I][B]Nguồn: MathScope.ORG[/B][/I][/RIGHT]
 
Thụy An is offline   Trả Lời Với Trích Dẫn
Old 19-10-2017, 06:05 PM   #8
babyteen9x
+Thành Viên+
 
Tham gia ngày: Jul 2010
Bài gởi: 8
Thanks: 4
Thanked 1 Time in 1 Post
Trích:
Nguyên văn bởi MATHSCOPE View Post

$\boxed{1}$ [Hà Nội] Cho $x;\, y;\, z$ là các số hữu tỉ sao cho $x+y^{2}+z^{2}$, $y+z^{2}+x^{2}$ và $z+x^{2}+y^{2}$ đều là các số nguyên. Chứng minh rằng $2x$ là số nguyên.
Ta viết $x = \dfrac{a}{m};\,y = \dfrac{b}{m};\,z = \dfrac{c}{m}$ với $a;\,b;\,c;\,m\in\mathbb Z,\;m>0$ và $\gcd(a;\,b;\,c;\,m)=1$, để có
\[\begin{array}{l}
ma + {b^2} + {c^2}\; \vdots \;{m^2}\\
mb + {c^2} + {a^2}\; \vdots \;{m^2}\\
mc + {a^2} + {b^2}\; \vdots \;{m^2}
\end{array}\]
Từ đây có $a^2+b^2;\,b^2+c^2;\,c^2+a^2$ đều là bội của $m$, do đó
\[2a^2=\left( {{c^2} + {a^2}} \right) + \left( {{a^2} + {b^2}} \right) - \left( {{b^2} + {c^2}} \right)\; \vdots \;m\]
Tương tự ta cũng có $m\mid 2b^2$ và $m\mid 2c^2$, xét hai trường hợp sau
  1. Nếu $m$ lẻ, ta có luôn $a^2;\,b^2;\,c^2$ đều là bội của $m$, mà $\gcd(a;\,b;\,c;\,m)=1$ cho nên $m=1$ tức $x\in\mathbb Z$ vì thế tất nhiên có điều cần chứng minh.
  2. Nếu $m$ chẵn, viết $m=2k$ ta có luôn $a^2;\,b^2;\,c^2$ đều là bội của $k$, mà $\gcd(a;\,b;\,c;\,2k)=1$ cho nên $k=1$ tức $m=2$ và có $2x=a\in\mathbb Z$.

[RIGHT][I][B]Nguồn: MathScope.ORG[/B][/I][/RIGHT]
 
babyteen9x is offline   Trả Lời Với Trích Dẫn
Old 19-10-2017, 07:25 PM   #9
Cutrone
+Thành Viên+
 
Tham gia ngày: Oct 2017
Bài gởi: 8
Thanks: 2
Thanked 0 Times in 0 Posts
Trích:
Nguyên văn bởi MATHSCOPE View Post
$\boxed{27}$ [Hà Tĩnh] Cho hai đa thức bậc ba:
\[P(x)=x^3+2x^2-7x-16,\quad Q(x)=x^3+3x^2+8x-4\]
  • Chứng minh rằng mỗi đa thức đều có một nghiệm dương duy nhất.
  • Gọi các nghiệm dương của $P(x),\, Q(x)$ lần lượt là $p;\, q$. Chứng minh rằng: \[\sqrt{p}-\sqrt{q}=1.\]
  1. Có $P(0)P(3)<0$ và $Q(0)Q(1)<0$, nên theo định lý hàm liên tục thì chắc chắn $P(x)$ và $Q(x)$ có các nghiệm thực dương.

    Nếu $P(x)$ có hai nghiệm thực dương, thế thì theo Viettè tích các nghiệm của $P(x)$ là 16 nên nó có cả ba nghiệm thực dương, nhưng điều này dẫn đến vô lý vì cũng theo Viettè thì tổng các nghiệm của $P(x)$ là -2.

    Tương tự, theo Viettè tổng các nghiệm của $Q(x)$ là -3 còn tích chúng là 4, nên $Q(x)$ không thể có hai nghiệm thực dương.

    Vậy, mỗi đa thức đã cho có và có duy nhất một nghiệm thực dương.
  2. Đặt $\sqrt q =r$ thế thì do $q$ là nghiệm thực dương của $Q(x)$ và $q=r^2$, nên $r$ là nghiệm thực dương của đa thức
    \[Q\left( {{x^2}} \right) = {x^6} + 3{x^4} + 8{x^2} - 4\]
    Để ý rằng
    \[Q\left( {{x^2}} \right) =\left( {{x^3} - {x^2} + 2x + 2} \right)\left( {{x^3} + {x^2} + 2x - 2} \right)\]
    Đồng thời lại có đánh giá
    \[{x^3} - {x^2} + 2x + 2 = x{\left( {x - 1} \right)^2} + {x^2} + x + 2 > 0\;\forall\,x>0\]
    Vậy nên có ${{r^3} + {r^2} + 2r - 2}=0 $, lại xét
    \[\begin{align*}
    P\left( {{{\left( {r + 1} \right)}^2}} \right) &= {\left( {r + 1} \right)^6} + 2{\left( {r + 1} \right)^2} - 7{\left( {r + 1} \right)^2} - 16\\
    &= {r^6} + 6{r^5} + 17{r^4} + 28{r^3} + 20{r^2} - 20\\
    &= \left( {{r^3} + {r^2} + 2r - 2} \right)\left( {{r^3} + 5{r^2} + 10r + 10} \right)\\
    &= 0
    \end{align*}\]
    Vậy $(r+1)^2$ là nghiệm dương của $P(x)$, tức $p=(r+1)^2=\left(1+\sqrt q\right)^2$, và ta có điều phải chứng minh sau khi lấy căn bậc hai.

[RIGHT][I][B]Nguồn: MathScope.ORG[/B][/I][/RIGHT]
 

thay đổi nội dung bởi: Cutrone, 19-10-2017 lúc 07:27 PM
Cutrone is offline   Trả Lời Với Trích Dẫn
Old 22-10-2017, 05:25 PM   #10
hoanganhtran
Moderator
 
Tham gia ngày: Jan 2017
Bài gởi: 8
Thanks: 1
Thanked 3 Times in 3 Posts
66 [Đắk Lắk] Tìm tất cả các bộ số nguyên $(a;b;c;d)$ thoả
$a^2+35=5^b6^c7^d$
Lời giải:
Giả sử tồn tại các số nguyên $(a,b,c,d)$ thỏa mãn đề bài. Hiển nhiên b,c,d phải là các số tự nhiên.
Nếu $b>1 \Rightarrow 5^b6^c7^d \equiv 0 (mod 5)$, lại có 35 chia hết cho 5 nên $a^2 \equiv 0 (mod 5) \Rightarrow a \equiv 0 (mod 5) \Rightarrow a^2 \equiv 0 (mod 25) \Rightarrow a^2+35 \equiv 10 (mod 25)$ mà $5^b6^c7^d \equiv 0 (mod 25)$ (mâu thuẫn) nên hoặc $b=0$ hoặc $b=1$.
CMTT ta có hoặc $d=0$ hoặc $d=1$
Trường hợp $b=0$ $d=1$ thì $ a^2+35=7.6^c$ mà $ 6^b \equiv 1 (mod 5) \Rightarrow 7.6^c \equiv 2 (mod 5) \Rightarrow a^2+35 \equiv 2 mod 5) \Rightarrow a^2 \equiv 2(mod 5)$ mà 1 số chính phương chỉa đồng dư 0,1,4 modun 5( mâu thuẫn)
Trường hợp $b=1$ $d=0$ thì $ a^2+35=5.6^c$ suy ra $a$ phải chia hết cho 5 nên $a^2 \equiv 0 (mod 25) \Rightarrow a^2+35 \equiv 10 (mod 25) \Rightarrow 5.6^c \equiv 10 (mod 25) \Rightarrow 6^c \equiv 2 (mod 5)$ ( vô lí do $6^c \equiv \pm1 (mod 7)$)
Trường hợp $ b=1$ $c=1$ suy ra $a$ chia hết cho 5 và 7. Đặt $a=35k$ với $k$ là nguyên suy ra $35k^2+1=6^c \Rightarrow 1 \equiv 6^c (mod 7) \Rightarrow c \equiv 0 (mod 2) $.
Nếu $c=0$ thì $k=0$
Nếu $c>0$ thì $35k^2+1$ là số chẵn nên $k$ lẻ suy ra $k^2 \equiv 1 (mod 8) \Rightarrow 35k^2+1 \equiv 4 (mod 8) \Rightarrow 6^c \equiv 4 (mod 8) \Rightarrow c=2 \Rightarrow k=1\Rightarrow a=35$
Trường hợp $b=c=0$ thì $a^2+35=6^c$
Do $6^c \geq 35 \Rightarrow c \geq 2 \Rightarrow a^2+35 \equiv 0 (mod 2)$ nên a lẻ suy ra $a^2+35 \equiv 4 (mod 8) \Rightarrow c=2 \Rightarrow a=1$
Kết luận no...
[RIGHT][I][B]Nguồn: MathScope.ORG[/B][/I][/RIGHT]
 
hoanganhtran is offline   Trả Lời Với Trích Dẫn
Old 22-10-2017, 06:03 PM   #11
hoanganhtran
Moderator
 
Tham gia ngày: Jan 2017
Bài gởi: 8
Thanks: 1
Thanked 3 Times in 3 Posts
4 [Đắk Lắk] Tìm số nguyên dương $n$ sao cho $(n^2+11n−4)n!+33.13^n+4$ là một số chính phương.
Lời giải:
Giả sử tồn tại n thỏa mãn đề bài. Khi đó tồn tại số tự nhiên $A$ sao cho $(n^2+11n−4)n!+33.13^n+4=A^2$
Xét $n \geq 4 \Rightarrow n! \equiv 0 (mod8)$
Nếu $n \equiv 0 (mod 2) \Rightarrow 13^n \equiv 1 (mod 8) \Rightarrow 33.13^n+4 \equiv 5 (mod 8) \Rightarrow A^2 \equiv 5 (mod 8)$ (vô lí do 1 số chính phương chỉ đồng dư 0,1,4 modun 8)
Vậy $n \geq 4$ thì n lẻ.
Ta có $ n^2+11n-4 \equiv n^2 -3n-4 \equiv (n-4)(n+1) \equiv (n+3)(n+1) (mod 7) \Rightarrow (n^2+11n-4)n! \equiv (n+3)(n+1)! (mod 7)$
Do $n \geq 4$ nên với $n$ khác 5 thì $(n+3)(n+1)! \equiv 0 (mod 7) \Rightarrow A^2 \equiv 33.13^n+4 \equiv (-2)(-1)+4=6 (mod 7)$ (do n lẻ)( vô lí do 1 số chính phương chỉ đồng dư 0,1,2,4 modun 7)
Vậy ta chỉ cần xét $n \in {1,2,3,5} $ để tìm $n$ thỏa mãn đề bài.
[RIGHT][I][B]Nguồn: MathScope.ORG[/B][/I][/RIGHT]
 

thay đổi nội dung bởi: hoanganhtran, 22-10-2017 lúc 11:05 PM
hoanganhtran is offline   Trả Lời Với Trích Dẫn
Old 22-10-2017, 06:33 PM   #12
hoanganhtran
Moderator
 
Tham gia ngày: Jan 2017
Bài gởi: 8
Thanks: 1
Thanked 3 Times in 3 Posts
7 [Hoà Bình] Cho $a;b;c$ là 3 số nguyên thỏa mãn
$$a+b+c=a^2(c−b)+b^2.(a−c)+c^2(b−a)$$
Chứng minh rằng $a+b+ca+b+c$ chia hết cho 27.
Lời giải:
Giả sử tồn tại các số nguyên $a;b;c$ thỏa mãn đề bài.
Ta có $$a+b+c=a^2(c−b)+b^2.(a−c)+c^2(b−a)=(a-b)(b-c)(c-a)$$
Nếu $a;b;c$ lập thành hệ thặng dư đầy đủ modun 3 thì $a+b+c \equiv 0 (mod 3) \Rightarrow (a-b)(b-c)(c-a) \equiv 0 (mod 3)$ nên tồn tại 2 trong 3 số đồng dư với nhau trong modun 3 nên $(a;b;c)$ không lập thành hệ thặng dư đầy đủ modun 3(mâu thuẫn)
Nếu $(a;b;c)$ không lập thành hệ thặng dư đầy đủ modun 3 tức là trong 3 số a;b;c có 2 số đồng dư với nhau theo modun 3 nên$$(a-b)(b-c)(c-a) \equiv 0 (mod 3) \Rightarrow a+b+c \equiv 0 (mod 3)$$.
Không mất tính tổng quát, giả sử $$a \equiv b (mod 3) \Rightarrow 2a+c \equiv 0 (mod 3) \Rightarrow a \equiv c (mod 3) \Rightarrow a-b \equiv b-c \equiv c-a \equiv 0 (mod 3) \Rightarrow (a-b)(b-c)(c-a) \equiv 0 (mod 27)$$
Suy ra $a+b+c$ chia hết cho 27(đpcm)
[RIGHT][I][B]Nguồn: MathScope.ORG[/B][/I][/RIGHT]
 
hoanganhtran is offline   Trả Lời Với Trích Dẫn
Old 22-10-2017, 08:56 PM   #13
thepduc
+Thành Viên+
 
Tham gia ngày: Jul 2017
Bài gởi: 5
Thanks: 1
Thanked 1 Time in 1 Post
Trích:
Nguyên văn bởi hoanganhtran View Post
Nếu $n \geq 3 \Rightarrow n! \equiv 0 (mod 3) \Rightarrow (n^2+11n−4)n!+33.13^n+4 \equiv 2 (mod 3)$
Mình nghĩ bạn Hoàng Anh nhầm chỗ này, phải là
$$(n^2+11n−4)n!+33.13^n+4 \equiv 1 \pmod 3 $$
[RIGHT][I][B]Nguồn: MathScope.ORG[/B][/I][/RIGHT]
 
thepduc is offline   Trả Lời Với Trích Dẫn
The Following User Says Thank You to thepduc For This Useful Post:
hoanganhtran (22-10-2017)
Old 22-10-2017, 09:17 PM   #14
Thụy An
+Thành Viên+

 
Tham gia ngày: Oct 2017
Bài gởi: 93
Thanks: 1
Thanked 68 Times in 45 Posts
Trích:
Nguyên văn bởi MATHSCOPE View Post
$\boxed{16}$ [Chuyên KHTN Hà Nội] Cho dãy số $\left(a_n\right)$ xác định bởi công thức sau: \[a_0=1;\,a_1=4;\,a_{n+2}=2a_{n+1}+3a_n\quad\forall \,n \in\mathbb N.\]
Chứng minh rằng trong dãy số trên không có số nào là bội của 2017.
Giải phương trình đặc trưng $x^2=2x+3$ để có công thức truy hồi của dãy là
\[{a_n} = \frac{{{{5.3}^n} - {{\left( { - 1} \right)}^n}}}{4}\quad\forall\,n\in\mathbb N.\]
Giả sử tồn tại số tự nhiên $n$ để $2017\mid a_n$, lúc đó có đồng dư
\[{5.3^n} \equiv {\left( { - 1} \right)^n}\pmod{2017};\;(*)\]
Để ý rằng $\left( {\dfrac{{ - 1}}{{2017}}} \right) = {\left( { - 1} \right)^{\frac{{2017 - 1}}{2}}} = 1$ , và theo luật thuận nghịch thì
\[\left( {\frac{3}{{2017}}} \right)\left( {\frac{{2017}}{3}} \right) = {\left( { - 1} \right)^{\frac{{\left( {3 - 1} \right)\left( {2017 - 1} \right)}}{4}}} = 1\]
Trong khi $2017^{\frac{3-1}{2}}\equiv 1\pmod 3$, cho nên chứng tỏ $3$ là thặng dư bậc hai theo mod $2017$.

Những điều đó kết hợp với $(*)$ cho ta $5$ là thặng dư bậc hai mod $2017$; $(1)$, đồng thời
\[\left( {\frac{5}{{2017}}} \right)\left( {\frac{{2017}}{5}} \right) = {\left( { - 1} \right)^{\frac{{\left( {5 - 1} \right)\left( {2017 - 1} \right)}}{4}}} = 1\]
Có điều, $2017^{\frac{5-1}{2}}\equiv -1\pmod 5$ nên
\[\left( {\frac{5}{{2017}}} \right) = \left( {\frac{{2017}}{5}} \right) = - 1;\;(2).\]
Từ mâu thuẫn của $(1)$ và $(2)$, cho ta điều cần chứng minh.
[RIGHT][I][B]Nguồn: MathScope.ORG[/B][/I][/RIGHT]
 
Thụy An is offline   Trả Lời Với Trích Dẫn
Old 23-10-2017, 04:51 PM   #15
hoanganhtran
Moderator
 
Tham gia ngày: Jan 2017
Bài gởi: 8
Thanks: 1
Thanked 3 Times in 3 Posts
20 [Hà Tĩnh] Tìm tất cả các cặp số nguyên $(a;b)$ sao cho với mọi số nguyên dương $n$, ta có $n$ chia hết cho $a^n+b^{n+1}$.
Lời giải:
Giả sử tồn tại bộ số nguyên dương $(a,b)$ thỏa mãn đề bài.
Ta thấy nếu một trong 2 số $a;b$ bằng 0 thì số còn lại là $ \in1$.
Xét trường hợp cả a và b đều khác 0.
Theo giả thiết, ta chọn $p$ là số nguyên tố và $p> |a|+|b^2|$ thì $p>a;b \Rightarrow (a,p)=(b,p)=1$ và $a^p+b^{p+1}$ là ước của $p \Rightarrow a^p+b^{p+1} \in {1;-1;p;-p}$
Do $(a,p)=(b,p)=1 \Rightarrow a^n+b^{n+1} \equiv a+b^2 (mod p) \Rightarrow (a^n+b^{n+1},p)=1$ (vì $a+b^2 <p$)
Suy ra $a^n+b^{n+1} \pm 1 $ nên $a;b$ khác tính chẵn lẻ hay $a^n+b^{n+1} \equiv 1 (mod 2) \forall n \in N$
Chọn $n=2^k$ với $k$ là số tự nhiên. Theo giả thiết ta có $a^{2^k}+b^{2^k+1}$ là ước của $2^k$ mà $a;b$ khác tính chẵn lẻ nên $a^{2^k}+b^{2^k+1}= \pm1 \forall k \in N*$
Suy ra tồn tại dãy số tự nhiên vô hạn S={$ k_1;k_2;... $}
sao cho $a^n+b^{n+1} =1 \forall n \in S$ hoặc $a^n+b^{n+1} =-1 \forall n \in S$
Trường hợp 1: $a^n+b^{n+1} =1 \forall n \in S$ hay $a^{2^{k_i}}+b^{2^{k_i}+1} =1 \forall n \in S$
Nếu $a$ lẻ thì $V_2(a^{2^(k_i)}-1)=V_2(b^{2^(k_i)+1} \Rightarrow V_2(a^2-1)+V_2(2^{k_i})-1=(2^{k_i}+1).V_2(b) \Rightarrow V_2(a^2-1)>2^{k_i}-k_i$, cho i chạy đến vô cùng thì $V_2(a^2-1)$ cũng tiến tới vô cùng nên $a= \pm 1$
Nếu $a$ chẵn ta có $V_2(b^{2^(k_i)+1}-1)=V_2(b-1)=V_2(a^{2^(k_i)})$ cho i chạy tới vô cùng( vô lí)
Trường hợp 2 làm tương tự.
Vậy 1 trong 2 số luôn có 1 số bằng $\pm 1$
Nếu $a=1$ thì $b^{n+1}+1$ là ước của n với mọi n. Chọn n bằng 1 thì $b=0$
Nếu $a=-1$ thì chọn n=1 thì b=0
Nếu $b=1$ thì chọn n=1 và n=2 suy ra a=0
Nếu $b=-1$ thì chọn n=1 và n=2 suy ra a=0.
[RIGHT][I][B]Nguồn: MathScope.ORG[/B][/I][/RIGHT]
 
hoanganhtran is offline   Trả Lời Với Trích Dẫn
The Following User Says Thank You to hoanganhtran For This Useful Post:
MATHSCOPE (23-10-2017)
Trả lời Gởi Ðề Tài Mới

Bookmarks


Quuyền Hạn Của Bạn
You may not post new threads
You may not post replies
You may not post attachments
You may not edit your posts

BB code is Mở
Smilies đang Mở
[IMG] đang Mở
HTML đang Tắt

Chuyển đến


Múi giờ GMT. Hiện tại là 01:43 AM.


Powered by: vBulletin Copyright ©2000-2024, Jelsoft Enterprises Ltd.
Inactive Reminders By mathscope.org
[page compression: 121.68 k/138.00 k (11.83%)]